Verification of Stoke's Theorem

  • Thread starter Thread starter sunnyday11
  • Start date Start date
  • Tags Tags
    Theorem
Click For Summary
The discussion focuses on verifying Stoke's Theorem for the vector field F = y i + z j + x k over the surface S defined by the paraboloid z = 1 - (x^2 + y^2). The line integral calculated using the parameterization r(t) = cos(t) i + sin(t) j resulted in -π, but there was a discrepancy with the flux of the curl of F. The curl of F was determined to be -(i + j + k), and the double integral was set up incorrectly, missing a "+1" in the integrand. The correct approach involves using polar coordinates and ensuring the differential is rdrdt. Accurate calculations are essential for confirming the theorem's validity.
sunnyday11
Messages
14
Reaction score
0

Homework Statement



Verify Stoke's Theorem for F = y i + z j + x k and S the paraboloid z=1-(x^2+y^2) with z bigger or equal to 0 oriented upward, and the curve C which is the boundary of S.



Homework Equations



Stoke's Theorem
line interal = flux of curl of F (don't know how to use the symbol function)

The Attempt at a Solution



For the line integral, I use r(t) = cos (t) i + sin (t) j , and the solution i got was -pi but I could not get the flux of curl of F to match.

Curl F = -(i+j+k)
double integral of (-2x-2y-1)dxdy
==> double integral of (-2r cost(t) - 2r sin(t) drdt with r [0,1] and t [0, 2pi]


Thank you very much!
 
Physics news on Phys.org
sunnyday11 said:

Homework Statement



Verify Stoke's Theorem for F = y i + z j + x k and S the paraboloid z=1-(x^2+y^2) with z bigger or equal to 0 oriented upward, and the curve C which is the boundary of S.



Homework Equations



Stoke's Theorem
line interal = flux of curl of F (don't know how to use the symbol function)

The Attempt at a Solution



For the line integral, I use r(t) = cos (t) i + sin (t) j , and the solution i got was -pi but I could not get the flux of curl of F to match.

Curl F = -(i+j+k)
double integral of (-2x-2y-1)dxdy
==> double integral of (-2r cost(t) - 2r sin(t) drdt with r [0,1] and t [0, 2pi]
I agree with you up to here but changing
\int\int (-2x- 2y+ 1)dxdy
to polar coordinates gives
\int\int (-2rcos(t)- 2r sin(t)+ 1)(rdrdt)


That is, you seem to have forgotten the "+ 1" in the integrand. And the differential in polar coordinates is rdrdt, not just "drdt".


Thank you very much!
 
Question: A clock's minute hand has length 4 and its hour hand has length 3. What is the distance between the tips at the moment when it is increasing most rapidly?(Putnam Exam Question) Answer: Making assumption that both the hands moves at constant angular velocities, the answer is ## \sqrt{7} .## But don't you think this assumption is somewhat doubtful and wrong?

Similar threads

  • · Replies 4 ·
Replies
4
Views
2K
  • · Replies 7 ·
Replies
7
Views
2K
  • · Replies 8 ·
Replies
8
Views
2K
  • · Replies 26 ·
Replies
26
Views
3K
  • · Replies 2 ·
Replies
2
Views
2K
Replies
6
Views
3K
  • · Replies 10 ·
Replies
10
Views
1K
  • · Replies 3 ·
Replies
3
Views
2K
  • · Replies 3 ·
Replies
3
Views
2K
  • · Replies 2 ·
Replies
2
Views
2K